Is $limsup_{nrightarrowinfty}sqrt[n]{|a_n|}=frac{1}{R}=limsup_{nrightarrowinfty}|frac{a_{n+1}}{a_n}|$?












1












$begingroup$


$R$ is the radius of convergence for a powerseries



I will write down my proof but I am not sure whether this is right because I thought $$limsup_{nrightarrowinfty}sqrt[n]{|a_n|}leq limsup_{nrightarrowinfty}|frac{a_{n+1}}{a_n}|$$ If I would take a sequence $(a_n)_{ninmathbb{N}}$ for which $$limsup_{nrightarrowinfty}sqrt[n]{|a_n|} < 1 <limsup_{nrightarrowinfty}|frac{a_{n+1}}{a_n}|$$ wouldn't there be a contradiction for the respective power series. Because $$limsup_{nrightarrowinfty}sqrt[n]{|a_n|} < limsup_{nrightarrowinfty}|frac{a_{n+1}}{a_n}|$$ and the statement in my question would imply $frac{1}{R}<frac{1}{R}$





Please tell me where I made the mistake in my reasoning of the following proof:



i) $frac{1}{R}=limsup_{nrightarrowinfty}sqrt[n]{|a_n|}=:tneq 0,infty$



Applying the root criteria for an arbitrary power series $sum_{n=0}^{infty}a_nz^n$



$$limsup_{nrightarrowinfty}sqrt[n]{|a_nz^n|}=limsup_{nrightarrowinfty}sqrt[n]{|a_n}||z|$$



Converges absolutely if



$$limsup_{nrightarrowinfty}sqrt[n]{|a_n}||z|<1iff |z|<frac{1}{limsup_{nrightarrowinfty}sqrt[n]{|a_n}|}=frac{1}{t}tag{*}$$



Diverges if



$$limsup_{nrightarrowinfty}sqrt[n]{|a_n}||z|>1iff |z|>frac{1}{limsup_{nrightarrowinfty}sqrt[n]{|a_n}|}=frac{1}{t}tag{**}$$



Suppose $frac{1}{R}>tiff R<frac{1}{t}Rightarrow R<frac{R+frac{1}{t}}{2}<frac{1}{t}$



$(*) Rightarrow frac{R+frac{1}{t}}{2}$, converges absolutely. Contradiction



Because $R:=sup{|z|:sum_{n=0}^{infty}a_nz^n$, converges$}$



Suppose $frac{1}{R}<t$, $(**) Rightarrow$ The power series $sum_{n=0}^{infty}a_nz^n$ diverges for $z=frac{R+frac{1}{t}}{2}$ Contradiction



Because $forall zin mathbb{C}: |z|<R, sum_{n=0}^{infty}a_nz^n $ converges absolutely.





ii) $frac{1}{R}=limsup_{nrightarrowinfty}|frac{a_{n+1}}{a_n}|=:tneq 0,infty$



Applying the quotient criteria for an arbitrary power series $sum_{n=0}^{infty}a_nz^n$



$$limsup_{nrightarrowinfty}|frac{a_{n+1}z^{n+1}}{a_nz^n}|iff limsup_{nrightarrowinfty}|frac{a_{n+1}}{a_n}||z| $$



Converges absolutely if



$$limsup_{nrightarrowinfty}|frac{a_{n+1}}{a_n}||z|<1iff|z|<frac{1}{limsup_{nrightarrowinfty}|frac{a_{n+1}}{a_n}|}= frac{1}{t}tag{***}$$



Diverges if



$$limsup_{nrightarrowinfty}|frac{a_{n+1}}{a_n}||z|>1iff|z|>frac{1}{limsup_{nrightarrowinfty}|frac{a_{n+1}}{a_n}|}= frac{1}{t}tag{****}$$



Suppose $frac{1}{R}>tiff R<frac{1}{t}Rightarrow R<frac{R+frac{1}{t}}{2}<frac{1}{t}$



$(***) Rightarrow frac{R+frac{1}{t}}{2}$, converges absolutely. Contradiction



Because $R:=sup{|z|:sum_{n=0}^{infty}a_nz^n$, converges$}$



Suppose $frac{1}{R}<t$, $(****) Rightarrow$ The powerseries $sum_{n=0}^{infty}a_nz^n$ diverges for $z=frac{R+frac{1}{t}}{2}$ Contradiction



Because $forall zin mathbb{C}: |z|<R, sum_{n=0}^{infty}a_nz^n $ converges absolutely.





i) + ii) $Rightarrow limsup_{nrightarrowinfty}sqrt[n]{|a_n|}=frac{1}{R}=limsup_{nrightarrowinfty}|frac{a_{n+1}}{a_n}|$



I am thinking about this for some time now please help me to solve the problem.










share|cite|improve this question











$endgroup$












  • $begingroup$
    You probably want to remove the $n$'th power of $a_n$ in each one of its twelve occurrences.
    $endgroup$
    – uniquesolution
    Dec 29 '18 at 17:22












  • $begingroup$
    This should help: en.wikipedia.org/wiki/…
    $endgroup$
    – Ben W
    Dec 29 '18 at 17:22










  • $begingroup$
    Yes I will remove them
    $endgroup$
    – RM777
    Dec 29 '18 at 17:22












  • $begingroup$
    What's the radius of convergence of $1+2x+x^2+2x^3+x^4+2x^5+cdots$?
    $endgroup$
    – Lord Shark the Unknown
    Dec 29 '18 at 17:27






  • 1




    $begingroup$
    See en.wikipedia.org/wiki/Ratio_test and look for inconclusive.
    $endgroup$
    – egreg
    Dec 29 '18 at 18:17


















1












$begingroup$


$R$ is the radius of convergence for a powerseries



I will write down my proof but I am not sure whether this is right because I thought $$limsup_{nrightarrowinfty}sqrt[n]{|a_n|}leq limsup_{nrightarrowinfty}|frac{a_{n+1}}{a_n}|$$ If I would take a sequence $(a_n)_{ninmathbb{N}}$ for which $$limsup_{nrightarrowinfty}sqrt[n]{|a_n|} < 1 <limsup_{nrightarrowinfty}|frac{a_{n+1}}{a_n}|$$ wouldn't there be a contradiction for the respective power series. Because $$limsup_{nrightarrowinfty}sqrt[n]{|a_n|} < limsup_{nrightarrowinfty}|frac{a_{n+1}}{a_n}|$$ and the statement in my question would imply $frac{1}{R}<frac{1}{R}$





Please tell me where I made the mistake in my reasoning of the following proof:



i) $frac{1}{R}=limsup_{nrightarrowinfty}sqrt[n]{|a_n|}=:tneq 0,infty$



Applying the root criteria for an arbitrary power series $sum_{n=0}^{infty}a_nz^n$



$$limsup_{nrightarrowinfty}sqrt[n]{|a_nz^n|}=limsup_{nrightarrowinfty}sqrt[n]{|a_n}||z|$$



Converges absolutely if



$$limsup_{nrightarrowinfty}sqrt[n]{|a_n}||z|<1iff |z|<frac{1}{limsup_{nrightarrowinfty}sqrt[n]{|a_n}|}=frac{1}{t}tag{*}$$



Diverges if



$$limsup_{nrightarrowinfty}sqrt[n]{|a_n}||z|>1iff |z|>frac{1}{limsup_{nrightarrowinfty}sqrt[n]{|a_n}|}=frac{1}{t}tag{**}$$



Suppose $frac{1}{R}>tiff R<frac{1}{t}Rightarrow R<frac{R+frac{1}{t}}{2}<frac{1}{t}$



$(*) Rightarrow frac{R+frac{1}{t}}{2}$, converges absolutely. Contradiction



Because $R:=sup{|z|:sum_{n=0}^{infty}a_nz^n$, converges$}$



Suppose $frac{1}{R}<t$, $(**) Rightarrow$ The power series $sum_{n=0}^{infty}a_nz^n$ diverges for $z=frac{R+frac{1}{t}}{2}$ Contradiction



Because $forall zin mathbb{C}: |z|<R, sum_{n=0}^{infty}a_nz^n $ converges absolutely.





ii) $frac{1}{R}=limsup_{nrightarrowinfty}|frac{a_{n+1}}{a_n}|=:tneq 0,infty$



Applying the quotient criteria for an arbitrary power series $sum_{n=0}^{infty}a_nz^n$



$$limsup_{nrightarrowinfty}|frac{a_{n+1}z^{n+1}}{a_nz^n}|iff limsup_{nrightarrowinfty}|frac{a_{n+1}}{a_n}||z| $$



Converges absolutely if



$$limsup_{nrightarrowinfty}|frac{a_{n+1}}{a_n}||z|<1iff|z|<frac{1}{limsup_{nrightarrowinfty}|frac{a_{n+1}}{a_n}|}= frac{1}{t}tag{***}$$



Diverges if



$$limsup_{nrightarrowinfty}|frac{a_{n+1}}{a_n}||z|>1iff|z|>frac{1}{limsup_{nrightarrowinfty}|frac{a_{n+1}}{a_n}|}= frac{1}{t}tag{****}$$



Suppose $frac{1}{R}>tiff R<frac{1}{t}Rightarrow R<frac{R+frac{1}{t}}{2}<frac{1}{t}$



$(***) Rightarrow frac{R+frac{1}{t}}{2}$, converges absolutely. Contradiction



Because $R:=sup{|z|:sum_{n=0}^{infty}a_nz^n$, converges$}$



Suppose $frac{1}{R}<t$, $(****) Rightarrow$ The powerseries $sum_{n=0}^{infty}a_nz^n$ diverges for $z=frac{R+frac{1}{t}}{2}$ Contradiction



Because $forall zin mathbb{C}: |z|<R, sum_{n=0}^{infty}a_nz^n $ converges absolutely.





i) + ii) $Rightarrow limsup_{nrightarrowinfty}sqrt[n]{|a_n|}=frac{1}{R}=limsup_{nrightarrowinfty}|frac{a_{n+1}}{a_n}|$



I am thinking about this for some time now please help me to solve the problem.










share|cite|improve this question











$endgroup$












  • $begingroup$
    You probably want to remove the $n$'th power of $a_n$ in each one of its twelve occurrences.
    $endgroup$
    – uniquesolution
    Dec 29 '18 at 17:22












  • $begingroup$
    This should help: en.wikipedia.org/wiki/…
    $endgroup$
    – Ben W
    Dec 29 '18 at 17:22










  • $begingroup$
    Yes I will remove them
    $endgroup$
    – RM777
    Dec 29 '18 at 17:22












  • $begingroup$
    What's the radius of convergence of $1+2x+x^2+2x^3+x^4+2x^5+cdots$?
    $endgroup$
    – Lord Shark the Unknown
    Dec 29 '18 at 17:27






  • 1




    $begingroup$
    See en.wikipedia.org/wiki/Ratio_test and look for inconclusive.
    $endgroup$
    – egreg
    Dec 29 '18 at 18:17
















1












1








1





$begingroup$


$R$ is the radius of convergence for a powerseries



I will write down my proof but I am not sure whether this is right because I thought $$limsup_{nrightarrowinfty}sqrt[n]{|a_n|}leq limsup_{nrightarrowinfty}|frac{a_{n+1}}{a_n}|$$ If I would take a sequence $(a_n)_{ninmathbb{N}}$ for which $$limsup_{nrightarrowinfty}sqrt[n]{|a_n|} < 1 <limsup_{nrightarrowinfty}|frac{a_{n+1}}{a_n}|$$ wouldn't there be a contradiction for the respective power series. Because $$limsup_{nrightarrowinfty}sqrt[n]{|a_n|} < limsup_{nrightarrowinfty}|frac{a_{n+1}}{a_n}|$$ and the statement in my question would imply $frac{1}{R}<frac{1}{R}$





Please tell me where I made the mistake in my reasoning of the following proof:



i) $frac{1}{R}=limsup_{nrightarrowinfty}sqrt[n]{|a_n|}=:tneq 0,infty$



Applying the root criteria for an arbitrary power series $sum_{n=0}^{infty}a_nz^n$



$$limsup_{nrightarrowinfty}sqrt[n]{|a_nz^n|}=limsup_{nrightarrowinfty}sqrt[n]{|a_n}||z|$$



Converges absolutely if



$$limsup_{nrightarrowinfty}sqrt[n]{|a_n}||z|<1iff |z|<frac{1}{limsup_{nrightarrowinfty}sqrt[n]{|a_n}|}=frac{1}{t}tag{*}$$



Diverges if



$$limsup_{nrightarrowinfty}sqrt[n]{|a_n}||z|>1iff |z|>frac{1}{limsup_{nrightarrowinfty}sqrt[n]{|a_n}|}=frac{1}{t}tag{**}$$



Suppose $frac{1}{R}>tiff R<frac{1}{t}Rightarrow R<frac{R+frac{1}{t}}{2}<frac{1}{t}$



$(*) Rightarrow frac{R+frac{1}{t}}{2}$, converges absolutely. Contradiction



Because $R:=sup{|z|:sum_{n=0}^{infty}a_nz^n$, converges$}$



Suppose $frac{1}{R}<t$, $(**) Rightarrow$ The power series $sum_{n=0}^{infty}a_nz^n$ diverges for $z=frac{R+frac{1}{t}}{2}$ Contradiction



Because $forall zin mathbb{C}: |z|<R, sum_{n=0}^{infty}a_nz^n $ converges absolutely.





ii) $frac{1}{R}=limsup_{nrightarrowinfty}|frac{a_{n+1}}{a_n}|=:tneq 0,infty$



Applying the quotient criteria for an arbitrary power series $sum_{n=0}^{infty}a_nz^n$



$$limsup_{nrightarrowinfty}|frac{a_{n+1}z^{n+1}}{a_nz^n}|iff limsup_{nrightarrowinfty}|frac{a_{n+1}}{a_n}||z| $$



Converges absolutely if



$$limsup_{nrightarrowinfty}|frac{a_{n+1}}{a_n}||z|<1iff|z|<frac{1}{limsup_{nrightarrowinfty}|frac{a_{n+1}}{a_n}|}= frac{1}{t}tag{***}$$



Diverges if



$$limsup_{nrightarrowinfty}|frac{a_{n+1}}{a_n}||z|>1iff|z|>frac{1}{limsup_{nrightarrowinfty}|frac{a_{n+1}}{a_n}|}= frac{1}{t}tag{****}$$



Suppose $frac{1}{R}>tiff R<frac{1}{t}Rightarrow R<frac{R+frac{1}{t}}{2}<frac{1}{t}$



$(***) Rightarrow frac{R+frac{1}{t}}{2}$, converges absolutely. Contradiction



Because $R:=sup{|z|:sum_{n=0}^{infty}a_nz^n$, converges$}$



Suppose $frac{1}{R}<t$, $(****) Rightarrow$ The powerseries $sum_{n=0}^{infty}a_nz^n$ diverges for $z=frac{R+frac{1}{t}}{2}$ Contradiction



Because $forall zin mathbb{C}: |z|<R, sum_{n=0}^{infty}a_nz^n $ converges absolutely.





i) + ii) $Rightarrow limsup_{nrightarrowinfty}sqrt[n]{|a_n|}=frac{1}{R}=limsup_{nrightarrowinfty}|frac{a_{n+1}}{a_n}|$



I am thinking about this for some time now please help me to solve the problem.










share|cite|improve this question











$endgroup$




$R$ is the radius of convergence for a powerseries



I will write down my proof but I am not sure whether this is right because I thought $$limsup_{nrightarrowinfty}sqrt[n]{|a_n|}leq limsup_{nrightarrowinfty}|frac{a_{n+1}}{a_n}|$$ If I would take a sequence $(a_n)_{ninmathbb{N}}$ for which $$limsup_{nrightarrowinfty}sqrt[n]{|a_n|} < 1 <limsup_{nrightarrowinfty}|frac{a_{n+1}}{a_n}|$$ wouldn't there be a contradiction for the respective power series. Because $$limsup_{nrightarrowinfty}sqrt[n]{|a_n|} < limsup_{nrightarrowinfty}|frac{a_{n+1}}{a_n}|$$ and the statement in my question would imply $frac{1}{R}<frac{1}{R}$





Please tell me where I made the mistake in my reasoning of the following proof:



i) $frac{1}{R}=limsup_{nrightarrowinfty}sqrt[n]{|a_n|}=:tneq 0,infty$



Applying the root criteria for an arbitrary power series $sum_{n=0}^{infty}a_nz^n$



$$limsup_{nrightarrowinfty}sqrt[n]{|a_nz^n|}=limsup_{nrightarrowinfty}sqrt[n]{|a_n}||z|$$



Converges absolutely if



$$limsup_{nrightarrowinfty}sqrt[n]{|a_n}||z|<1iff |z|<frac{1}{limsup_{nrightarrowinfty}sqrt[n]{|a_n}|}=frac{1}{t}tag{*}$$



Diverges if



$$limsup_{nrightarrowinfty}sqrt[n]{|a_n}||z|>1iff |z|>frac{1}{limsup_{nrightarrowinfty}sqrt[n]{|a_n}|}=frac{1}{t}tag{**}$$



Suppose $frac{1}{R}>tiff R<frac{1}{t}Rightarrow R<frac{R+frac{1}{t}}{2}<frac{1}{t}$



$(*) Rightarrow frac{R+frac{1}{t}}{2}$, converges absolutely. Contradiction



Because $R:=sup{|z|:sum_{n=0}^{infty}a_nz^n$, converges$}$



Suppose $frac{1}{R}<t$, $(**) Rightarrow$ The power series $sum_{n=0}^{infty}a_nz^n$ diverges for $z=frac{R+frac{1}{t}}{2}$ Contradiction



Because $forall zin mathbb{C}: |z|<R, sum_{n=0}^{infty}a_nz^n $ converges absolutely.





ii) $frac{1}{R}=limsup_{nrightarrowinfty}|frac{a_{n+1}}{a_n}|=:tneq 0,infty$



Applying the quotient criteria for an arbitrary power series $sum_{n=0}^{infty}a_nz^n$



$$limsup_{nrightarrowinfty}|frac{a_{n+1}z^{n+1}}{a_nz^n}|iff limsup_{nrightarrowinfty}|frac{a_{n+1}}{a_n}||z| $$



Converges absolutely if



$$limsup_{nrightarrowinfty}|frac{a_{n+1}}{a_n}||z|<1iff|z|<frac{1}{limsup_{nrightarrowinfty}|frac{a_{n+1}}{a_n}|}= frac{1}{t}tag{***}$$



Diverges if



$$limsup_{nrightarrowinfty}|frac{a_{n+1}}{a_n}||z|>1iff|z|>frac{1}{limsup_{nrightarrowinfty}|frac{a_{n+1}}{a_n}|}= frac{1}{t}tag{****}$$



Suppose $frac{1}{R}>tiff R<frac{1}{t}Rightarrow R<frac{R+frac{1}{t}}{2}<frac{1}{t}$



$(***) Rightarrow frac{R+frac{1}{t}}{2}$, converges absolutely. Contradiction



Because $R:=sup{|z|:sum_{n=0}^{infty}a_nz^n$, converges$}$



Suppose $frac{1}{R}<t$, $(****) Rightarrow$ The powerseries $sum_{n=0}^{infty}a_nz^n$ diverges for $z=frac{R+frac{1}{t}}{2}$ Contradiction



Because $forall zin mathbb{C}: |z|<R, sum_{n=0}^{infty}a_nz^n $ converges absolutely.





i) + ii) $Rightarrow limsup_{nrightarrowinfty}sqrt[n]{|a_n|}=frac{1}{R}=limsup_{nrightarrowinfty}|frac{a_{n+1}}{a_n}|$



I am thinking about this for some time now please help me to solve the problem.







sequences-and-series complex-analysis limsup-and-liminf






share|cite|improve this question















share|cite|improve this question













share|cite|improve this question




share|cite|improve this question








edited Dec 30 '18 at 11:04









LutzL

60.9k42157




60.9k42157










asked Dec 29 '18 at 17:17









RM777RM777

38312




38312












  • $begingroup$
    You probably want to remove the $n$'th power of $a_n$ in each one of its twelve occurrences.
    $endgroup$
    – uniquesolution
    Dec 29 '18 at 17:22












  • $begingroup$
    This should help: en.wikipedia.org/wiki/…
    $endgroup$
    – Ben W
    Dec 29 '18 at 17:22










  • $begingroup$
    Yes I will remove them
    $endgroup$
    – RM777
    Dec 29 '18 at 17:22












  • $begingroup$
    What's the radius of convergence of $1+2x+x^2+2x^3+x^4+2x^5+cdots$?
    $endgroup$
    – Lord Shark the Unknown
    Dec 29 '18 at 17:27






  • 1




    $begingroup$
    See en.wikipedia.org/wiki/Ratio_test and look for inconclusive.
    $endgroup$
    – egreg
    Dec 29 '18 at 18:17




















  • $begingroup$
    You probably want to remove the $n$'th power of $a_n$ in each one of its twelve occurrences.
    $endgroup$
    – uniquesolution
    Dec 29 '18 at 17:22












  • $begingroup$
    This should help: en.wikipedia.org/wiki/…
    $endgroup$
    – Ben W
    Dec 29 '18 at 17:22










  • $begingroup$
    Yes I will remove them
    $endgroup$
    – RM777
    Dec 29 '18 at 17:22












  • $begingroup$
    What's the radius of convergence of $1+2x+x^2+2x^3+x^4+2x^5+cdots$?
    $endgroup$
    – Lord Shark the Unknown
    Dec 29 '18 at 17:27






  • 1




    $begingroup$
    See en.wikipedia.org/wiki/Ratio_test and look for inconclusive.
    $endgroup$
    – egreg
    Dec 29 '18 at 18:17


















$begingroup$
You probably want to remove the $n$'th power of $a_n$ in each one of its twelve occurrences.
$endgroup$
– uniquesolution
Dec 29 '18 at 17:22






$begingroup$
You probably want to remove the $n$'th power of $a_n$ in each one of its twelve occurrences.
$endgroup$
– uniquesolution
Dec 29 '18 at 17:22














$begingroup$
This should help: en.wikipedia.org/wiki/…
$endgroup$
– Ben W
Dec 29 '18 at 17:22




$begingroup$
This should help: en.wikipedia.org/wiki/…
$endgroup$
– Ben W
Dec 29 '18 at 17:22












$begingroup$
Yes I will remove them
$endgroup$
– RM777
Dec 29 '18 at 17:22






$begingroup$
Yes I will remove them
$endgroup$
– RM777
Dec 29 '18 at 17:22














$begingroup$
What's the radius of convergence of $1+2x+x^2+2x^3+x^4+2x^5+cdots$?
$endgroup$
– Lord Shark the Unknown
Dec 29 '18 at 17:27




$begingroup$
What's the radius of convergence of $1+2x+x^2+2x^3+x^4+2x^5+cdots$?
$endgroup$
– Lord Shark the Unknown
Dec 29 '18 at 17:27




1




1




$begingroup$
See en.wikipedia.org/wiki/Ratio_test and look for inconclusive.
$endgroup$
– egreg
Dec 29 '18 at 18:17






$begingroup$
See en.wikipedia.org/wiki/Ratio_test and look for inconclusive.
$endgroup$
– egreg
Dec 29 '18 at 18:17












1 Answer
1






active

oldest

votes


















2












$begingroup$

Your reading of the quotient criterion is partly wrong. The rule for divergence is that if
$$
liminf_{ntoinfty}frac{|a_{n+1}z^{n+1}|}{|a_nz^n|}=|z|liminf_{ntoinfty}frac{|a_{n+1}|}{|a_n|}>1
$$

then the power series diverges. This gives you a third radius to consider. Thus you get
$$
R_{quot, sup}le R_{root, sup}le R_{quot,inf}
$$

and only the radius $R_{root, sup}$ of the root criterion gives the exact region of convergence of the power series. If the coefficient quotients to not have a strict limit, the quotient criterion leaves out an annulus where no claim towards convergence or divergence is possible.






share|cite|improve this answer









$endgroup$














    Your Answer








    StackExchange.ready(function() {
    var channelOptions = {
    tags: "".split(" "),
    id: "69"
    };
    initTagRenderer("".split(" "), "".split(" "), channelOptions);

    StackExchange.using("externalEditor", function() {
    // Have to fire editor after snippets, if snippets enabled
    if (StackExchange.settings.snippets.snippetsEnabled) {
    StackExchange.using("snippets", function() {
    createEditor();
    });
    }
    else {
    createEditor();
    }
    });

    function createEditor() {
    StackExchange.prepareEditor({
    heartbeatType: 'answer',
    autoActivateHeartbeat: false,
    convertImagesToLinks: true,
    noModals: true,
    showLowRepImageUploadWarning: true,
    reputationToPostImages: 10,
    bindNavPrevention: true,
    postfix: "",
    imageUploader: {
    brandingHtml: "Powered by u003ca class="icon-imgur-white" href="https://imgur.com/"u003eu003c/au003e",
    contentPolicyHtml: "User contributions licensed under u003ca href="https://creativecommons.org/licenses/by-sa/3.0/"u003ecc by-sa 3.0 with attribution requiredu003c/au003e u003ca href="https://stackoverflow.com/legal/content-policy"u003e(content policy)u003c/au003e",
    allowUrls: true
    },
    noCode: true, onDemand: true,
    discardSelector: ".discard-answer"
    ,immediatelyShowMarkdownHelp:true
    });


    }
    });














    draft saved

    draft discarded


















    StackExchange.ready(
    function () {
    StackExchange.openid.initPostLogin('.new-post-login', 'https%3a%2f%2fmath.stackexchange.com%2fquestions%2f3056054%2fis-limsup-n-rightarrow-infty-sqrtna-n-frac1r-limsup-n-rightarro%23new-answer', 'question_page');
    }
    );

    Post as a guest















    Required, but never shown

























    1 Answer
    1






    active

    oldest

    votes








    1 Answer
    1






    active

    oldest

    votes









    active

    oldest

    votes






    active

    oldest

    votes









    2












    $begingroup$

    Your reading of the quotient criterion is partly wrong. The rule for divergence is that if
    $$
    liminf_{ntoinfty}frac{|a_{n+1}z^{n+1}|}{|a_nz^n|}=|z|liminf_{ntoinfty}frac{|a_{n+1}|}{|a_n|}>1
    $$

    then the power series diverges. This gives you a third radius to consider. Thus you get
    $$
    R_{quot, sup}le R_{root, sup}le R_{quot,inf}
    $$

    and only the radius $R_{root, sup}$ of the root criterion gives the exact region of convergence of the power series. If the coefficient quotients to not have a strict limit, the quotient criterion leaves out an annulus where no claim towards convergence or divergence is possible.






    share|cite|improve this answer









    $endgroup$


















      2












      $begingroup$

      Your reading of the quotient criterion is partly wrong. The rule for divergence is that if
      $$
      liminf_{ntoinfty}frac{|a_{n+1}z^{n+1}|}{|a_nz^n|}=|z|liminf_{ntoinfty}frac{|a_{n+1}|}{|a_n|}>1
      $$

      then the power series diverges. This gives you a third radius to consider. Thus you get
      $$
      R_{quot, sup}le R_{root, sup}le R_{quot,inf}
      $$

      and only the radius $R_{root, sup}$ of the root criterion gives the exact region of convergence of the power series. If the coefficient quotients to not have a strict limit, the quotient criterion leaves out an annulus where no claim towards convergence or divergence is possible.






      share|cite|improve this answer









      $endgroup$
















        2












        2








        2





        $begingroup$

        Your reading of the quotient criterion is partly wrong. The rule for divergence is that if
        $$
        liminf_{ntoinfty}frac{|a_{n+1}z^{n+1}|}{|a_nz^n|}=|z|liminf_{ntoinfty}frac{|a_{n+1}|}{|a_n|}>1
        $$

        then the power series diverges. This gives you a third radius to consider. Thus you get
        $$
        R_{quot, sup}le R_{root, sup}le R_{quot,inf}
        $$

        and only the radius $R_{root, sup}$ of the root criterion gives the exact region of convergence of the power series. If the coefficient quotients to not have a strict limit, the quotient criterion leaves out an annulus where no claim towards convergence or divergence is possible.






        share|cite|improve this answer









        $endgroup$



        Your reading of the quotient criterion is partly wrong. The rule for divergence is that if
        $$
        liminf_{ntoinfty}frac{|a_{n+1}z^{n+1}|}{|a_nz^n|}=|z|liminf_{ntoinfty}frac{|a_{n+1}|}{|a_n|}>1
        $$

        then the power series diverges. This gives you a third radius to consider. Thus you get
        $$
        R_{quot, sup}le R_{root, sup}le R_{quot,inf}
        $$

        and only the radius $R_{root, sup}$ of the root criterion gives the exact region of convergence of the power series. If the coefficient quotients to not have a strict limit, the quotient criterion leaves out an annulus where no claim towards convergence or divergence is possible.







        share|cite|improve this answer












        share|cite|improve this answer



        share|cite|improve this answer










        answered Dec 30 '18 at 11:10









        LutzLLutzL

        60.9k42157




        60.9k42157






























            draft saved

            draft discarded




















































            Thanks for contributing an answer to Mathematics Stack Exchange!


            • Please be sure to answer the question. Provide details and share your research!

            But avoid



            • Asking for help, clarification, or responding to other answers.

            • Making statements based on opinion; back them up with references or personal experience.


            Use MathJax to format equations. MathJax reference.


            To learn more, see our tips on writing great answers.




            draft saved


            draft discarded














            StackExchange.ready(
            function () {
            StackExchange.openid.initPostLogin('.new-post-login', 'https%3a%2f%2fmath.stackexchange.com%2fquestions%2f3056054%2fis-limsup-n-rightarrow-infty-sqrtna-n-frac1r-limsup-n-rightarro%23new-answer', 'question_page');
            }
            );

            Post as a guest















            Required, but never shown





















































            Required, but never shown














            Required, but never shown












            Required, but never shown







            Required, but never shown

































            Required, but never shown














            Required, but never shown












            Required, but never shown







            Required, but never shown







            Popular posts from this blog

            Le Mesnil-Réaume

            Ida-Boy-Ed-Garten

            web3.py web3.isConnected() returns false always